Resumen
A continuación demostraremos que una densidad Lagrangiana compatible para el campo electromagnético en el espacio vacío es \begin{equation} \mathcal{L}_{em}\:=\:\epsilon_{0}\cdot\dfrac{\left|\!\left|\mathbf{E}\right|\!\right|^{2}-c^{2}\left|\!\left|\mathbf{B}\right|\!\right|^{2}}{2}-\rho \phi + \mathbf{j} \boldsymbol{\cdot} \mathbf{A} \tag{045} \end{equation} es decir, las ecuaciones de Euler-Langrange producidas a partir de esta Lagrangiana son las ecuaciones de Maxwell para el campo electromagnético.
Esta densidad lagrangiana se obtiene por tanteo (1) procedimiento, no adivinando.
1. Introducción
Las ecuaciones diferenciales de Maxwell del campo electromagnético en el espacio vacío son \begin{align} \boldsymbol{\nabla} \boldsymbol{\times} \mathbf{E} & = -\frac{\partial \mathbf{B}}{\partial t} \tag{001a}\\ \boldsymbol{\nabla} \boldsymbol{\times} \mathbf{B} & = \mu_{0}\mathbf{j}+\frac{1}{c^{2}}\frac{\partial \mathbf{E}}{\partial t} \tag{001b}\\ \nabla \boldsymbol{\cdot} \mathbf{E} & = \frac{\rho}{\epsilon_{0}} \tag{001c}\\ \nabla \boldsymbol{\cdot}\mathbf{B}& = 0 \tag{001d} \end{align} donde $\: \mathbf{E} =$ vector de intensidad del campo eléctrico, $\:\mathbf{B}=$ vector de densidad de flujo magnético, $\:\rho=$ densidad de carga eléctrica, $\:\mathbf{j} =$ vector de densidad de corriente eléctrica. Todas las magnitudes son funciones de las tres coordenadas espaciales $\:\left( x_{1},x_{2},x_{3}\right) \equiv \left( x,y,z\right)\:$ y el tiempo $\:t \equiv x_{4}\:$ .
A partir de la ecuación (001d) el vector flujo magnético $\:\mathbf{B}\:$ puede expresarse como el rizo de un potencial vectorial $\:\mathbf{A}\:$ \begin{equation} \mathbf{B}=\boldsymbol{\nabla} \boldsymbol{\times} \mathbf{A} \tag{002} \end{equation} y de (002) la ecuación (001a) resulta \begin{equation} \boldsymbol{\nabla} \boldsymbol{\times}\left(\mathbf{E}+\frac{\partial \mathbf{A}}{\partial t}\right) =\boldsymbol{0} \tag{003} \end{equation} por lo que el término entre paréntesis puede expresarse como el gradiente de una función escalar \begin{equation*} \mathbf{E}+\frac{\partial \mathbf{A}}{\partial t} =-\boldsymbol{\nabla}\phi \end{equation*} es decir \begin{equation} \mathbf{E} =-\boldsymbol{\nabla}\phi -\frac{\partial \mathbf{A}}{\partial t} \tag{004} \end{equation} Así que las seis variables escalares, los componentes de los vectores $\:\mathbf{E}\:$ y $\:\mathbf{B}\:$ pueden expresarse como funciones de 4 variables escalares, el potencial escalar $\:\phi\:$ y tres componentes del potencial vectorial $\:\mathbf{A}$ .
Inserción de las expresiones de $\:\mathbf{E}\:$ y $\:\mathbf{B}\:$ , ecuaciones (002) y (004) respectivamente, en las ecuaciones (001b) y (001c) tenemos
\begin{equation} \boldsymbol{\nabla} \boldsymbol{\times} \left(\boldsymbol{\nabla} \boldsymbol{\times} \mathbf{A}\right) =\mu_{0}\mathbf{j}+\frac{1}{c^{2}}\frac{\partial }{\partial t}\left(-\boldsymbol{\nabla}\phi -\frac{\partial \mathbf{A}}{\partial t}\right) \tag{005} \end{equation}
y \begin{equation} \boxed{\: -\nabla^{2}\phi-\frac{\partial }{\partial t}\left(\nabla \boldsymbol{\cdot}\mathbf{A}\right) =\frac{\rho}{\epsilon_{0}} \:} \tag{006} \end{equation} Dado que \begin{equation} \boldsymbol{\nabla} \boldsymbol{\times} \left( \boldsymbol{\nabla} \boldsymbol{\times} \mathbf{A}\right) =\boldsymbol{\nabla}\left(\nabla \boldsymbol{\cdot}\mathbf{A}\right)- \nabla^{2}\mathbf{A} \tag{007} \end{equation} la ecuación (005) da como resultado \begin{equation} \boxed{\: \frac{1}{c^{2}}\frac{\partial^{2}\mathbf{A}}{\partial t^{2}}-\nabla^{2}\mathbf{A}+ \boldsymbol{\nabla}\left(\nabla \boldsymbol{\cdot} \mathbf{A}+\frac{1}{c^{2}}\frac{\partial \phi}{\partial t}\right) =\mu_{0}\mathbf{j} \:} \tag{008} \end{equation}
2. Las ecuaciones de Euler-Lagrange del campo EM
Ahora, nuestra tarea principal es encontrar una densidad lagrangiana $\:\mathcal{L}\:$ función de las cuatro ''coordenadas de campo'' y sus derivadas de primer orden
\begin{equation} \mathcal{L}=\mathcal{L}\left(\eta_{\jmath}, \overset{\centerdot}{\eta}_{\jmath}, \boldsymbol{\nabla}\eta_{\jmath}\right) \qquad \left(\jmath=1,2,3,4\right) \tag{009} \end{equation} de forma que las cuatro ecuaciones escalares de campo electromagnético (006) y (008) se derivan de las ecuaciones de Lagrange \begin{equation} \frac{\partial }{\partial t}\left[\frac{\partial \mathcal{L}}{\partial \left(\dfrac{\partial \eta_{\jmath}}{\partial t}\right)}\right]+\sum_{k=1}^{k=3}\frac{\partial }{\partial x_{k}}\left[\frac{\partial \mathcal{L}}{\partial \left(\dfrac{\partial \eta_{\jmath}}{\partial x_{k}}\right)}\right]- \frac{\partial \mathcal{L}}{\partial \eta_{\jmath}}=0\:, \quad \left(\jmath=1,2,3,4\right) \tag{010} \end{equation} simplificado en notación a \begin{equation} \boxed{\: \dfrac{\partial }{\partial t}\left(\dfrac{\partial \mathcal{L}}{\partial \overset{\centerdot}{\eta}_{\jmath}}\right) + \nabla \boldsymbol{\cdot}\left[\dfrac{\partial \mathcal{L}}{\partial \left(\boldsymbol{\nabla}\eta_{\jmath}\right)}\right]- \frac{\partial \mathcal{L}}{\partial \eta_{\jmath}}=0, \quad \left(\jmath=1,2,3,4\right) \:} \tag{011} \end{equation}
Aquí la densidad lagrangiana $\:\mathcal{L}\:$ es función de
- las cuatro ''coordenadas de campo''
\begin{align} \eta_{1}&=\mathrm{A}_1\left( x_{1},x_{2},x_{3},t\right) \tag{012.1}\\ \eta_{2}&=\mathrm{A}_2\left( x_{1},x_{2},x_{3},t\right) \tag{012.2}\\ \eta_{3}&=\mathrm{A}_3\left( x_{1},x_{2},x_{3},t\right) \tag{012.3}\\ \eta_{4}&=\:\;\phi \left( x_{1},x_{2},x_{3},t\right) \tag{012.4} \end{align}
- sus derivadas temporales
\begin{align} \overset{\centerdot}{\eta}_{1} & \equiv \dfrac{\partial \eta_{1}}{\partial t} =\dfrac{\partial \mathrm{A}_{1}}{\partial t}\equiv\overset{\centerdot}{\mathrm{A}}_{1} \tag{013.1}\\ \overset{\centerdot}{\eta}_{2} & \equiv \dfrac{\partial \eta_{2}}{\partial t} =\dfrac{\partial \mathrm{A}_{2}}{\partial t}\equiv \overset{\centerdot}{\mathrm{A}}_{2} \tag{013.2}\\ \overset{\centerdot}{\eta}_{3} & \equiv \dfrac{\partial \eta_{3}}{\partial t} =\dfrac{\partial \mathrm{A}_{3}}{\partial t}\equiv\overset{\centerdot}{\mathrm{A}}_{3} \tag{013.3}\\ \overset{\centerdot}{\eta}_{4} & \equiv \dfrac{\partial \eta_{4}}{\partial t} =\dfrac{\partial \phi}{\partial t}\equiv\overset{\centerdot}{\phi} \tag{013.4} \end{align}
y
- sus gradientes
\begin{equation} \begin{array}{cccc} \boldsymbol{\nabla}\eta_{1}=\boldsymbol{\nabla}\mathrm{A}_1 \:,\: & \boldsymbol{\nabla}\eta_{2}=\boldsymbol{\nabla}\mathrm{A}_{2} \:,\: \boldsymbol{\nabla}\eta_{3}=\boldsymbol{\nabla}\mathrm{A}_3 \:,\: & \boldsymbol{\nabla}\eta_{4}=\boldsymbol{\nabla}\phi \end{array} \tag{014} \end{equation}
Expresamos las ecuaciones (006) y (008) en formas similares a las ecuaciones de Lagrange (011) \begin{equation} \boxed{\: \dfrac{\partial }{\partial t}\left(\nabla \boldsymbol{\cdot} \mathbf{A}\right)+\nabla \boldsymbol{\cdot} \left(\boldsymbol{\nabla}\phi \right) -\left(-\frac{\rho}{\epsilon_{0}}\right) =0 \:} \tag{015} \end{equation} y \begin{equation} \boxed{\: \dfrac{\partial}{\partial t}\left(\frac{\partial \mathrm{A}_{k}}{\partial t}+\frac{\partial \phi}{\partial x_{k}}\right)+\nabla \boldsymbol{\cdot} \left[c^{2}\left(\frac{\partial \mathbf{A}}{\partial x_{k}}- \boldsymbol{\nabla}\mathrm{A}_{k}\right)\right] -\frac{\mathrm{j}_{k}}{\epsilon_{0}}=0 \:} \tag{016} \end{equation} La ecuación de Lagrange (011) para $\:\jmath=4\:$ es decir, para $\:\eta_{4}=\phi \:$ es \begin{equation} \frac{\partial }{\partial t}\left(\frac{\partial \mathcal{L}}{\partial \overset{\centerdot}{\phi}}\right) + \nabla \boldsymbol{\cdot} \left[\frac{\partial \mathcal{L}}{\partial \left(\boldsymbol{\nabla}\phi\right)}\right]- \frac{\partial \mathcal{L}}{\partial \phi}=0 \tag{017} \end{equation}
Comparando las ecuaciones (015) y (017), observamos que la primera podría derivarse de la segunda si \begin{equation} \dfrac{\partial \mathcal{L}}{\partial \overset{\centerdot}{\phi}}=\nabla \boldsymbol{\cdot} \mathbf{A}\:, \qquad \dfrac{\partial \mathcal{L}}{\partial \left(\boldsymbol{\nabla}\phi\right)}=\boldsymbol{\nabla}\phi \:, \qquad \frac{\partial \mathcal{L}}{\partial \phi}=-\frac{\rho}{\epsilon_{0}} \tag{018} \end{equation}
de modo que la densidad lagrangiana $\:\mathcal{L}\:$ deben contener respectivamente los términos \begin{equation} \mathcal{L}_{\boldsymbol{\alpha_{1}}}\equiv\left(\nabla \boldsymbol{\cdot} \mathbf{A}\right)\overset{\centerdot}{\phi}\:, \qquad \mathcal{L}_{\boldsymbol{\alpha_{2}}}\equiv\tfrac{1}{2}\Vert \boldsymbol{\nabla}\phi \Vert^{2}\:, \qquad \mathcal{L}_{\boldsymbol{\alpha_{3}}}\equiv-\frac{\rho \phi}{\epsilon_{0}} \tag{019} \end{equation} y, en consecuencia, su suma \begin{equation} \mathcal{L}_{\boldsymbol{\alpha}}=\mathcal{L}_{\boldsymbol{\alpha_{1}}}+\mathcal{L}_{\boldsymbol{\alpha_{2}}} +\mathcal{L}_{\boldsymbol{\alpha_{3}}}=\left(\nabla \boldsymbol{\cdot} \mathbf{A}\right)\overset{\centerdot}{\phi}+\tfrac{1}{2}\Vert \boldsymbol{\nabla}\phi \Vert^{2}-\frac{\rho \phi}{\epsilon_{0}} \tag{020} \end{equation}
Suponemos que una densidad lagrangiana adecuada $\:\mathcal{L}\:$ sería de la forma \begin{equation} \mathcal{L}=\mathcal{L}_{\boldsymbol{\alpha}}+\mathcal{L}_{\boldsymbol{\beta}} \tag{021} \end{equation} y puesto que $\:\mathcal{L}_{\boldsymbol{\alpha}}\:$ produce la ecuación (015), esperamos que $\:\mathcal{L}_{\boldsymbol{\beta}}\:$ por determinar, dará lugar a las ecuaciones (016). Esta expectativa sería correcta si las ecuaciones (015) y (016) estuvieran desacopladas, por ejemplo si la primera contuviera $\:\phi $ -y el segundo $\:\mathbf{A} $ -sólo términos. Pero este no es el caso: $\:\mathcal{L}_{\boldsymbol{\alpha}}\:$ que contiene $\:\mathbf{A} $ -participarían en la producción de las ecuaciones (016) y además $\:\mathcal{L}_{\boldsymbol{\beta}}\:$ participaría en la producción de la ecuación (015), posiblemente destruyendo mutuamente la producción de las ecuaciones como esperábamos. Pero aquí seguimos un procedimiento de ensayo y error, que nos conducirá a la respuesta correcta como veremos a continuación.
Ahora, las ecuaciones de Lagrange (011) para $\:\jmath=k=1,2,3\:$ es decir, para $\:\eta_{k}=\mathrm{A}_{k} \:$ son \begin{equation} \frac{\partial }{\partial t}\left(\dfrac{\partial \mathcal{L}}{\partial \overset{\centerdot}{\mathrm{A}}_{k}}\right) + \nabla \boldsymbol{\cdot} \left[\dfrac{\partial \mathcal{L}}{\partial \left(\boldsymbol{\nabla}\mathrm{A}_{k}\right)}\right]- \frac{\partial \mathcal{L}}{\partial \mathrm{A}_{k}}=0 \tag{022} \end{equation}
Comparando las ecuaciones (016) y (022), observamos que la primera podría derivarse de la segunda si \begin{equation} \dfrac{\partial \mathcal{L}}{\partial \overset{\centerdot}{\mathrm{A}}_{k}}= \overset{\centerdot}{\mathrm{A}}_{k}+\frac{\partial \phi}{\partial x_{k}}\:, \quad \dfrac{\partial \mathcal{L}}{\partial \left(\boldsymbol{\nabla}\mathrm{A}_{k}\right)}=c^{2}\left(\frac{\partial \mathbf{A}}{\partial x_{k}}- \boldsymbol{\nabla}\mathrm{A}_{k}\right)\:, \quad \frac{\partial \mathcal{L}}{\partial \mathrm{A}}_{k}=\frac{\mathrm{j}_{k}}{\epsilon_{0}} \tag{023} \end{equation}
De la primera de las ecuaciones (023) se deduce que $\:\mathcal{L}_{\boldsymbol{\beta}}\:$ parte de la densidad de Lagrange $\:\mathcal{L}\:$ debe contener los términos \begin{equation} \frac{1}{2}\left\Vert \overset{\centerdot}{\mathrm{A}}_{k}\right\Vert^{2}+\frac{\partial \phi}{\partial x_{k}}\overset{\centerdot}{\mathrm{A}}_{k}\:, \quad k=1,2,3 \tag{024} \end{equation} por lo que su suma con respecto a $\:k\:$ \begin{equation} \mathcal{L}_{\boldsymbol{\beta_{1}}}\equiv \tfrac{1}{2}\left\Vert \mathbf{\dot{A}}\right\Vert^{2}+\boldsymbol{\nabla}\phi \boldsymbol{\cdot} \mathbf{\dot{A}} \tag{025} \end{equation}
A partir de la 2ª de las ecuaciones (023) el $\:\mathcal{L}_{\boldsymbol{\beta}}\:$ parte de la densidad de Lagrange $\:\mathcal{L}\:$ debe contener los términos \begin{equation} \tfrac{1}{2}c^{2}\left[\frac{\partial \mathbf{A}}{\partial x_{k}} \boldsymbol{\cdot} \boldsymbol{\nabla}\mathrm{A}_{k} -\Vert \boldsymbol{\nabla}\mathrm{A}_{k}\Vert^{2}\right] \:, \quad k=1,2,3 \tag{026} \end{equation} por lo que su suma con respecto a $\:k\:$ \begin{equation} \mathcal{L}_{\boldsymbol{\beta_{2}}}\equiv\tfrac{1}{2}c^{2}\sum^{k=3}_{k=1}\left[ \frac{\partial \mathbf{A}}{\partial x_{k}}\boldsymbol{\cdot} \boldsymbol{\nabla}\mathrm{A}_{k}-\Vert\boldsymbol{\nabla}\mathrm{A}_{k}\Vert^{2}\right] \tag{027} \end{equation} A partir de la 3ª de las ecuaciones (023), el $\:\mathcal{L}_{\boldsymbol{\beta}}\:$ parte de la densidad de Lagrange $\:\mathcal{L}\:$ debe contener los términos \begin{equation} \frac{\mathrm{j}_{k}\mathrm{A}_{k}}{\epsilon_{0}} \:, \quad k=1,2,3 \tag{028} \end{equation} por lo que su suma con respecto a $\:k\:$ \begin{equation} \mathcal{L}_{\boldsymbol{\beta_{3}}}\equiv \frac{\mathbf{j} \boldsymbol{\cdot} \mathbf{A}}{\epsilon_{0}} \tag{029} \end{equation}
A partir de las ecuaciones (025), (027) y (029) el $\:\mathcal{L}_{\boldsymbol{\beta}}\:$ parte de la densidad de Lagrange $\:\mathcal{L}\:$ es \begin{align} \mathcal{L}_{\boldsymbol{\beta}} & = \mathcal{L}_{\boldsymbol{\beta_{1}}}+\mathcal{L}_{\boldsymbol{\beta_{2}}} +\mathcal{L}_{\boldsymbol{\beta_{3}}} \tag{030}\\ & = \tfrac{1}{2}\left\Vert \mathbf{\dot{A}}\right\Vert^{2}+\boldsymbol{\nabla}\phi \boldsymbol{\cdot}\mathbf{\dot{A}}+\tfrac{1}{2}c^{2}\sum^{k=3}_{k=1}\left[ \frac{\partial \mathbf{A}}{\partial x_{k}} \boldsymbol{\cdot} \boldsymbol{\nabla}\mathrm{A}_{k}-\Vert\boldsymbol{\nabla}\mathrm{A}_{k}\Vert^{2}\right]+\frac{\mathbf{j} \boldsymbol{\cdot} \mathbf{A}}{\epsilon_{0}} \nonumber \end{align}
Por último, a partir de las expresiones (020) y (030) para las densidades $\:\mathcal{L}_{\boldsymbol{\alpha}},\mathcal{L}_{\boldsymbol{\beta}}\:$ la densidad de Lagrange $\:\mathcal{L}=\mathcal{L}_{\boldsymbol{\alpha}}+\mathcal{L}_{\boldsymbol{\beta}}\:$ es \begin{align} \mathcal{L}& = \mathcal{L}_{\boldsymbol{\alpha}} + \mathcal{L}_{\boldsymbol{\beta}} \tag{031}\\ & = \left(\nabla \boldsymbol{\cdot} \mathbf{A}\right)\overset{\centerdot}{\phi}+\tfrac{1}{2}\Vert \boldsymbol{\nabla}\phi \Vert^{2}-\frac{\rho \phi}{\epsilon_{0}} \nonumber\\ & + \tfrac{1}{2}\left\Vert \mathbf{\dot{A}}\right\Vert^{2}+\boldsymbol{\nabla}\phi \boldsymbol{\cdot} \mathbf{\dot{A}}+\tfrac{1}{2}c^{2}\sum^{k=3}_{k=1}\left[ \frac{\partial \mathbf{A}}{\partial x_{k}} \boldsymbol{\cdot} \boldsymbol{\nabla}\mathrm{A}_{k}-\Vert\boldsymbol{\nabla}\mathrm{A}_{k}\Vert^{2}\right]+\frac{\mathbf{j}\boldsymbol{\cdot}\mathbf{A}}{\epsilon_{0}} \nonumber\\ & \text{(this is a wrong Lagrange density)} \nonumber \end{align}
3. Error-prueba-éxito final
Inserción de esta expresión de densidad de Lagrange en la ecuación de Lagrange con respecto a $\:\phi \:$ es decir, la ecuación (017), no produce la ecuación (006) sino
\begin{equation} -\nabla^{2}\phi-\frac{\partial }{\partial t}\left(2\nabla \boldsymbol{\cdot} \mathbf{A}\right) =\frac{\rho}{\epsilon_{0}}\:, \quad (\textbf{wrong}) \tag{032} \end{equation} La aparición de un extra $\:\left( \nabla \boldsymbol{\cdot} \mathbf{A}\right) \:$ se debe al término $\:\left( \boldsymbol{\nabla}\phi \boldsymbol{\cdot} \mathbf{\dot{A}}\right) \:$ de $\:\mathcal{L}_{\boldsymbol{\beta}}\:$ y por eso la densidad de Lagrange dada por la ecuación (031) no es la adecuada.
Para resolver este problema debemos considerar (015), es decir (006), desde un punto de vista diferente, como sigue \begin{equation} \nabla \boldsymbol{\cdot}\left(\boldsymbol{\nabla}\phi + \mathbf{\dot{A}}\right) -\left(-\frac{\rho}{\epsilon_{0}}\right) =0 \tag{033} \end{equation}
Comparando las ecuaciones (033) y (017), observamos que la primera podría derivarse de la segunda si en lugar de (018) tenemos
\begin{equation} \dfrac{\partial \mathcal{L}}{\partial \overset{\centerdot}{\phi}}=0\:, \qquad \dfrac{\partial \mathcal{L}}{\partial \left(\boldsymbol{\nabla}\phi\right)}=\boldsymbol{\nabla}\phi + \mathbf{\dot{A}} \:, \qquad \frac{\partial \mathcal{L}}{\partial \phi}=-\frac{\rho}{\epsilon_{0}} \tag{034} \end{equation}
por lo que en lugar de (019) y (020) respectivamente las ecuaciones \begin{equation} \mathcal{L}^{\prime}_{\boldsymbol{\alpha_{1}}}\equiv 0\:, \quad \mathcal{L}^{\prime}_{\boldsymbol{\alpha_{2}}}\equiv\tfrac{1}{2}\Vert \boldsymbol{\nabla}\phi \Vert^{2} +\boldsymbol{\nabla}\phi \boldsymbol{\cdot} \mathbf{\dot{A}}\:, \quad \mathcal{L}^{\prime}_{\boldsymbol{\alpha_{3}}}=\mathcal{L}_{\boldsymbol{\alpha_{3}}}\equiv-\frac{\rho \phi}{\epsilon_{0}} \tag{035} \end{equation} \begin{equation} \mathcal{L}^{\prime}_{\boldsymbol{\alpha}}=\mathcal{L}^{\prime}_{\boldsymbol{\alpha_{1}}}+\mathcal{L}^{\prime}_{\boldsymbol{\alpha_{2}}} +\mathcal{L}^{\prime}_{\boldsymbol{\alpha_{3}}}=\tfrac{1}{2}\Vert \boldsymbol{\nabla}\phi \Vert^{2}+\boldsymbol{\nabla}\phi \boldsymbol{\cdot} \mathbf{\dot{A}}-\frac{\rho \phi}{\epsilon_{0}} \tag{036} \end{equation} Ahora, es necesario omitir de $\:\mathcal{L}_{\boldsymbol{\beta_{1}}}\:$ En la ecuación (025), el segundo término $\:\left( \boldsymbol{\nabla}\phi \boldsymbol{\cdot} \mathbf{\dot{A}}\right) \:$ ya que aparece en $\:\mathcal{L}^{\prime}_{\boldsymbol{\alpha_{2}}} \:$ véase la segunda de las ecuaciones anteriores (035).
Así que tenemos en lugar de (025) \begin{equation} \mathcal{L}^{\prime}_{\boldsymbol{\beta_{1}}}\equiv \tfrac{1}{2}\left\Vert \mathbf{\dot{A}}\right\Vert^{2} \tag{037} \end{equation} mientras que $\:\mathcal{L}_{\boldsymbol{\beta_{2}}},\mathcal{L}_{\boldsymbol{\beta_{3}}}\:$ permanecen invariables como en las ecuaciones (027) y (029) \begin{align} \mathcal{L}^{\prime}_{\boldsymbol{\beta_{2}}} &=\mathcal{L}_{\boldsymbol{\beta_{2}}}\equiv\tfrac{1}{2}c^{2}\sum^{k=3}_{k=1}\left[ \frac{\partial \mathbf{A}}{\partial x_{k}}\boldsymbol{\cdot}\boldsymbol{\nabla}\mathrm{A}_{k}-\Vert\boldsymbol{\nabla}\mathrm{A}_{k}\Vert^{2}\right] \tag{038} \\ \mathcal{L}^{\prime}_{\boldsymbol{\beta_{3}}} &=\mathcal{L}_{\boldsymbol{\beta_{3}}}\equiv \frac{\mathbf{j} \boldsymbol{\cdot} \mathbf{A}}{\epsilon_{0}} \tag{039} \end{align}
En lugar de (030) \begin{align} \mathcal{L}^{\prime}_{\boldsymbol{\beta}} & = \mathcal{L}^{\prime}_{\boldsymbol{\beta_{1}}}+\mathcal{L}^{\prime}_{\boldsymbol{\beta_{2}}} +\mathcal{L}^{\prime}_{\boldsymbol{\beta_{3}}} \tag{040} \\ & = \tfrac{1}{2}\left\Vert \mathbf{\dot{A}}\right\Vert^{2}+\tfrac{1}{2}c^{2}\sum^{k=3}_{k=1}\left[ \frac{\partial \mathbf{A}}{\partial x_{k}} \boldsymbol{\cdot} \boldsymbol{\nabla}\mathrm{A}_{k}-\Vert\boldsymbol{\nabla}\mathrm{A}_{k}\Vert^{2}\right]+\frac{\mathbf{j} \boldsymbol{\cdot} \mathbf{A}}{\epsilon_{0}} \nonumber \end{align} y finalmente para la nueva densidad lagrangiana tenemos en lugar de (031)
\begin{align} \mathcal{L}^{\prime}& = \mathcal{L}^{\prime}_{\boldsymbol{\alpha}} + \mathcal{L}^{\prime}_{\boldsymbol{\beta}} \tag{041} \\ & = \tfrac{1}{2}\Vert \boldsymbol{\nabla}\phi \Vert^{2} +\boldsymbol{\nabla}\phi \boldsymbol{\cdot} \mathbf{\dot{A}} -\frac{\rho \phi}{\epsilon_{0}} \nonumber\\ & + \tfrac{1}{2}\left\Vert \mathbf{\dot{A}}\right\Vert^{2}+\tfrac{1}{2}c^{2}\sum^{k=3}_{k=1}\left[ \frac{\partial \mathbf{A}}{\partial x_{k}} \boldsymbol{\cdot} \boldsymbol{\nabla}\mathrm{A}_{k} -\Vert\boldsymbol{\nabla}\mathrm{A}_{k}\Vert^{2}\right]+\frac{\mathbf{j} \boldsymbol{\cdot} \mathbf{A}}{\epsilon_{0}} \nonumber \end{align}
Densidad $\:\mathcal{L}^{\prime}\:$ de (041) se obtiene a partir de la densidad $\:\mathcal{L}\:$ de (031) si omitimos el término $\:\left(\nabla \boldsymbol{\cdot} \mathbf{A}\right)\overset{\centerdot}{\phi}\:$ . Así que $\:\mathcal{L}^{\prime}\:$ es independiente de $\:\overset{\centerdot}{\phi}$ .
En las siguientes ecuaciones el corchete sobre los 3 términos de la izquierda agrupa la parte de la densidad $\:\mathcal{L}^{\prime}\:$ que participa esencialmente a la producción de la ecuación electromagnética (006) a partir de la ecuación de Lagrange con respecto a $\:\phi \:$ ecuación (017), mientras que el corchete bajo los 4 términos de la derecha agrupa la parte de la densidad $\:\mathcal{L}^{\prime}\:$ que participa esencialmente a la producción de las ecuaciones electromagnéticas (008) a partir de las ecuaciones de Lagrange con respecto a $\:\mathrm{A}_{1},\mathrm{A}_{2},\mathrm{A}_{3} \:$ ecuación (022).
\begin{equation*} \mathcal{L}^{\prime}=\overbrace{\tfrac{1}{2}\Vert \boldsymbol{\nabla}\phi \Vert^{2}-\frac{\rho \phi}{\epsilon_{0}}+\boldsymbol{\nabla}\phi \boldsymbol{\cdot} \mathbf{\dot{A}}}^{\text{with respect to }\phi}+\tfrac{1}{2}\left\Vert \mathbf{\dot{A}}\right\Vert^{2}+\tfrac{1}{2}c^{2}\sum^{k=3}_{k=1}\left[\frac{\partial \mathbf{A}}{\partial x_{k}} \boldsymbol{\cdot} \boldsymbol{\nabla}\mathrm{A}_{k}-\Vert \boldsymbol{\nabla}\mathrm{A}_{k}\Vert^{2}\right]+\frac{\mathbf{j} \boldsymbol{\cdot} \mathbf{A}}{\epsilon_{0}} \end{equation*}
\begin{equation*} \mathcal{L}^{\prime}=\tfrac{1}{2}\Vert \boldsymbol{\nabla}\phi \Vert^{2}-\frac{\rho \phi}{\epsilon_{0}}+\underbrace{\boldsymbol{\nabla}\phi\boldsymbol{\cdot} \mathbf{\dot{A}}+\tfrac{1}{2}\left\Vert \mathbf{\dot{A}}\right\Vert^{2}+\tfrac{1}{2}c^{2}\sum^{k=3}_{k=1}\left[\frac{\partial \mathbf{A}}{\partial x_{k}} \boldsymbol{\cdot} \boldsymbol{\nabla}\mathrm{A}_{k}-\Vert\boldsymbol{\nabla}\mathrm{A}_{k}\Vert^{2}\right]+\frac{\mathbf{j}\boldsymbol{\cdot} \mathbf{A}}{\epsilon_{0}}}_{\text{with respect to }\mathbf{A}} \end{equation*}
Obsérvese el término común $\:\left( \boldsymbol{\nabla}\phi \boldsymbol{\cdot} \mathbf{\dot{A}}\right)$ .
Reordenación de los términos en la expresión (041) de la densidad $\:\mathcal{L}^{\prime}\:$ tenemos \begin{equation} \mathcal{L}^{\prime}=\underbrace{\tfrac{1}{2}\left\Vert \mathbf{\dot{A}}\right\Vert^{2}+\tfrac{1}{2}\Vert \boldsymbol{\nabla}\phi \Vert^{2}+\boldsymbol{\nabla}\phi \boldsymbol{\cdot} \mathbf{\dot{A}}}_{\tfrac{1}{2}\left\Vert - \boldsymbol{\nabla}\phi -\frac{\partial \mathbf{A}}{\partial t}\right\Vert^{2}}-\tfrac{1}{2}c^{2}\underbrace{\sum^{k=3}_{k=1}\left[\Vert \boldsymbol{\nabla}\mathrm{A}_{k}\Vert^{2}-\frac{\partial \mathbf{A}}{\partial x_{k}}\boldsymbol{\cdot} \boldsymbol{\nabla}\mathrm{A}_{k}\right]}_{\left\Vert \boldsymbol{\nabla} \boldsymbol{\times} \mathbf{A}\right\Vert^{2}}+\frac{1}{\epsilon_{0}}\left( -\rho \phi + \mathbf{j}\boldsymbol{\cdot} \mathbf{A}\right) \end{equation} \begin{equation} ----------------- \tag{042} \end{equation}
es decir \begin{equation} \mathcal{L}^{\prime}=\tfrac{1}{2}\left|\!\left|- \boldsymbol{\nabla}\phi -\frac{\partial \mathbf{A}}{\partial t}\right|\!\right|^{2}-\tfrac{1}{2}c^{2}\left|\!\left| \boldsymbol{\nabla} \boldsymbol{\times} \mathbf{A}\right|\!\right|^{2}+\frac{1}{\epsilon_{0}}\left( -\rho \phi + \mathbf{j} \boldsymbol{\cdot} \mathbf{A}\right) \tag{043} \end{equation} ou \begin{equation} \mathcal{L}^{\prime}=\frac{\left|\!\left|\mathbf{E}\right|\!\right|^{2}-c^{2}\left|\!\left|\mathbf{B}\right|\!\right|^{2}}{2}+\frac{1}{\epsilon_{0}}\left( -\rho \phi + \mathbf{j}\boldsymbol{\cdot}\mathbf{A}\right) \tag{044} \end{equation}
Ahora bien, si la densidad $\:\mathcal{L}^{\prime}\:$ debe tener dimensiones de energía por unidad de volumen que definimos $\:\mathcal{L}_{em}=\epsilon_{0}\mathcal{L}^{\prime} \:$ así que \begin{equation} \boxed{\: \mathcal{L}_{em}\:=\:\epsilon_{0}\cdot\dfrac{\left|\!\left|\mathbf{E}\right|\!\right|^{2}-c^{2}\left|\!\left|\mathbf{B}\right|\!\right|^{2}}{2}-\rho \phi + \mathbf{j} \boldsymbol{\cdot} \mathbf{A} \:} \tag{045} \end{equation} teniendo en cuenta que \begin{align} \left\Vert\mathbf{E}\right\Vert^{2} & = \left\Vert - \boldsymbol{\nabla}\phi -\dfrac{\partial \mathbf{A}}{\partial t}\right\Vert^{2} = \left\Vert \mathbf{\dot{A}}\right\Vert^{2}+\Vert \boldsymbol{\nabla}\phi \Vert^{2}+2\left(\boldsymbol{\nabla}\phi \boldsymbol{\cdot} \mathbf{\dot{A}}\right) \tag{046a}\\ & \nonumber\\ \left\Vert\mathbf{B}\right\Vert^{2} & = \left\Vert\boldsymbol{\nabla} \boldsymbol{\times} \mathbf{A}\right\Vert^{2}=\sum^{k=3}_{k=1}\left[\Vert \boldsymbol{\nabla}\mathrm{A}_{k}\Vert^{2}-\dfrac{\partial \mathbf{A}}{\partial x_{k}}\boldsymbol{\cdot} \boldsymbol{\nabla}\mathrm{A}_{k}\right] \tag{046b} \end{align}
El escalar $\:\left(\left|\!\left|\mathbf{E}\right|\!\right|^{2}-c^{2}\left|\!\left|\mathbf{B}\right|\!\right|^{2}\right)\:$ es uno de los dos invariantes de Lorentz (2) del campo (el otro es $\:\mathbf{E}\boldsymbol{\cdot}\mathbf{B}$ ) esencialmente igual a una constante veces $\:\mathcal{E}_{\mu\nu}\mathcal{E}^{\mu\nu}\:$ donde $\:\mathcal{E}^{\mu\nu}\:$ el campo antisimétrico (2) tensor.
Por otra parte, el escalar $\: \left(-\rho \phi + \mathbf{j} \boldsymbol{\cdot} \mathbf{A}\right)\:$ es esencialmente el producto interior $\:J_{\mu}A^{\mu}\:$ en el espacio de Minkowski de dos 4 vectores : la densidad de 4 corrientes $\:J^{\mu}=\left(c\rho,\mathbf{j}\right)\:$ y el 4-potencial $\:A^{\mu}=\left(\phi/c,\mathbf{A}\right)\:$ También es un escalar invariante de Lorentz.
Así, la densidad de Lagrange $\:\mathcal{L}_{em}\:$ en la ecuación (045) es invariante de Lorentz.
(1) Por un procedimiento de ensayo y error he encontrado el Lagrangiano en un caso más difícil y complicado : ver mi respuesta como user82794 aquí Obtener la Lagrangiana a partir del sistema de ecuaciones acopladas
(2) Siguiendo a W.Rindler en "Introducción a la Relatividad Especial" Ed.1982, este tensor se deriva en la ecuación (38.15) \begin{equation} \mathcal{E}_{\mu\nu}= \begin{bmatrix} 0 & E_{1} & E_{2} & E_{3} \\ -E_{1} & 0 & -cB_{3} & cB_{2} \\ -E_{2} & cB_{3} & 0 & -cB_{1} \\ -E_{3} & -cB_{2} & cB_{1} & 0 \end{bmatrix} \ \cuadrado \ es decir \begin{bmatrix} 0 & -E_{1} & - E_{2} & -E_{3} \\ E_{1} & 0 & -cB_{3} & cB_{2} \\ E_{2} & cB_{3} & 0 & -cB_{1} \\ E_{3} & -cB_{2} & cB_{1} & 0 \end{bmatrix} \38.15 \end{ecuación} que haciendo las sustituciones (de dualidad) $\:\mathbf{E}\to -c\mathbf{B}\:$ y $\:c\mathbf{B}\to \mathbf{E}\:$ produce \begin{equation} \mathcal{B}_{\mu\nu}= \begin{bmatrix} 0 & -cB_{1} & -cB_{2} & - cB_{3} \\ cB_{1} & 0 & -E_{3} & E_{2} \\ cB_{2} & cE_{3} & 0 & -E_{1} \\ cB_{3} & -E_{2} & E_{1} & 0 \end{bmatrix} \quad \text{so} \cuadrado \mathcal{B}^{\mu \begin{bmatrix} 0 & cB_{1} & cB_{2} & cB_{3} \\ -cB_{1} & 0 & -E_{3} & E_{2} \\ -cB_{2} & cE_{3} & 0 & -E_{1} \\ -cB_{3} & -E_{2} & E_{1} & 0 \end{bmatrix} \39,05 \end{ecuación} Las dos invariantes de $\:\mathcal{E}^{\mu\nu}\:$ -inmediatamente reconocibles como tales por su modo de formación- pueden expresarse del siguiente modo: \begin{align} X & =\dfrac{1}{2}\mathcal{E}_{\mu\nu}\mathcal{E}^{\mu\nu}=-\dfrac{1}{2}\mathcal{B}_{\mu\nu}\mathcal{B}^{\mu\nu}=c^{2}\left|\!\left|\mathbf{B}\right|\!\right|^{2}-\left|\!\left|\mathbf{E}\right|\!\right|^{2} \tag{39.06}\\ Y & =\dfrac{1}{4}\mathcal{B}_{\mu\nu}\mathcal{E}^{\mu\nu}=c\mathbf{B}\boldsymbol{\cdot}\mathbf{E} \tag{39.07} \end{align}
4 votos
¿De qué principios quiere partir?
0 votos
Relacionado: physics.stackexchange.com/q/20353/2451